ChaseDream

标题: 求助大家!!!prep12的两个题目 [打印本页]

作者: hnzjbenson    时间: 2012-9-9 14:57
标题: 求助大家!!!prep12的两个题目
Q1:
The OLEX Petroleum Company has recently determined that it could cut its refining costs by closing its refinery and consolidating all refining at its Tasberg refinery. Closing the Grenville refinery, however, would mean the immediate loss of about 1,200 jobs in the area. Eventually the lives of more than 10,000 people would be seriously disrupted. Therefore, OLEX's decision, announced yesterday, to keep open shows that at OLEX social concerns sometimes outweigh the desire for higher profits.

Which of the following, if true, most seriously undermines the argument given?

A. The Grenville refinery, although it operates at a higher cost than the Tasberg refinery, has nevertheless been moderately profitable for many years.
B. Even though OLEX could consolidate all its refining at the Tasberg plant, doing so at the Grenville plant would not be feasible.
C. The Tasberg refinery is more favorably situated than the refinery with respect to the major supply routes for raw petroleum.
D. If the Grenville refinery were ever closed and operations at the Tasberg refinery expanded, job openings at Tasberg would to the extent possible be filled with people formerly employed at
E. Closure of the Grenville refinery would mean compliance, at enormous cost, with demanding local codes regulating the cleanup of abandoned industrial sites.

The suggested answer is E,but I feel like D is a better choice than E. The argument of the question is that the company claims that its social concerns outweigh its desire for profits, so the argument is kind of comparison between social concerns against pursuit of profits.

For choice D, if employee formly employed at Grenville refinery would be hired back at Tasberg refinery, then there is no social concerns for closure of Grenville refinery. As a result, the argument stated by the company is actually a lie, and this should weaken the argument.

For choice E, the enormous costs of closure of Grenville refinery proves the non-sense of doing so, but this does not weaken the argument because you can not say that the company is not outweighing the social concerns against profit concern.

Q2:
A product that represents a clear technological advance over competing products can generally command a high price. Surprisingly, perhaps, the strategy to maximize overall profit from a new product is to charge than the greatest price the market will bear. Many companies charge the maximum possible price for such a product, because they want to make as much profit as they can and technological advances tend to be quickly surpassed. The drawback is that large profits on the new product give competitors a strong incentive to quickly match the new product's capabilities.

In the argument above, the two portions in boldface play which of the following roles?

(A) The first is the position the argument advocates; the second presents grounds for rejecting an alternative position.

(B) The first is the position the argument advocates; the second is an alternative pisition that argument rejects.

(C) The first is presents a strategy for achieving a certain goal; the second presents a drawback to that strategy.
(D) The first is a strategy for achieving a certain goal; the second presents grounds for preferring a different goal.

(E) The first is presents a strategy that, according to the argument, is ineffective; the second presents a way of improving the effectiveness of that strategy.

Why the suggested answer is A, but not C?

Many thanks!

作者: fionalovesunny    时间: 2013-5-27 06:05
Q1 支持D
作者: 漪阙夕缘    时间: 2013-7-21 15:01
同问Q2的C啊啊啊啊啊!!
作者: 晓野的野    时间: 2013-9-18 22:43
漪阙夕缘 发表于 2013-7-21 15:01
同问Q2的C啊啊啊啊啊!!

应该是在than前面有个less
这样的话就是两个策略了
一个是小于最大 一个是最大
作者: ALEXLOVELIFE    时间: 2014-8-9 08:41
同问,第二题为什么不选C啊




欢迎光临 ChaseDream (https://forum.chasedream.com/) Powered by Discuz! X3.3